nima1376

New Member
ارسال ها
218
لایک ها
93
امتیاز
0
#61
پاسخ : ماراتن نظریه ی اعداد (سطح پیشرفته)

یه سوال برای ادامه ی تاپیک

همه ی x,y های طبیعی را بیابید که x^2=y^3+23
طرفین رو با 4 جمع کنید

---- دو نوشته به هم متصل شده است ----

ثابت کنید مجموع ارقام توان های 9 از جایی به بعد بزرگتر مساوی 18 است
 

amirhossein1376

Well-Known Member
ارسال ها
284
لایک ها
272
امتیاز
63
#62
پاسخ : ماراتن نظریه ی اعداد (سطح پیشرفته)

راه حلتونو متوجه نمیشم من اینو با یه راه دیگه پیدا کردم ولی میشه بیشتر در مورد راه حلتون توضیح بدین؟

من اینو پیدا کردم که به شکل 4k+3 میشه و ازون به بعدش راحته
 

amirhossein1376

Well-Known Member
ارسال ها
284
لایک ها
272
امتیاز
63
#63
پاسخ : ماراتن نظریه ی اعداد (سطح پیشرفته)

سوال بعدی : تمام اعداد صحیح nبزرگتر از 1 رو پیدا کنید که این عدد صحیح شه:
 

nima1376

New Member
ارسال ها
218
لایک ها
93
امتیاز
0
#64
پاسخ : ماراتن نظریه ی اعداد (سطح پیشرفته)

سوال جهانی نبوده ؟؟
به راحتی با مرتبه حل میشه .....
 

amirhossein1376

Well-Known Member
ارسال ها
284
لایک ها
272
امتیاز
63
#65
پاسخ : ماراتن نظریه ی اعداد (سطح پیشرفته)

بله خوب به همین خاطرم گذاشتمش تو پیشرفته ولی جهانیه

---- دو نوشته به هم متصل شده است ----

اگر a,b طبیعی باشند نشان دهید اگر
عدد
را عاد کند آنگاه
منتقل شد به بخش ممتاز
 
آخرین ویرایش توسط مدیر

math1998

New Member
ارسال ها
336
لایک ها
224
امتیاز
0
#66
پاسخ : ماراتن نظریه ی اعداد (سطح پیشرفته)

تمام جواب های x , y را در اعداد طبیعی بدست اورید .

x[SUP]2y[/SUP]+ (x+1)[SUP]2y[/SUP][SUB]=(x+2)[SUP]2y[/SUP][/SUB]
 

math1998

New Member
ارسال ها
336
لایک ها
224
امتیاز
0
#68
پاسخ : ماراتن نظریه ی اعداد (سطح پیشرفته)

ثابت کنید بی نهایت عدد صحیح مثبت وجود دارد که n[SUP]2[/SUP]+1 یک عامل اول بزرگتر از 2n+(2n)[SUP]1/2[/SUP] دارد.
 

Armin_sf

New Member
ارسال ها
84
لایک ها
21
امتیاز
0
#69
پاسخ : ماراتن نظریه ی اعداد (سطح پیشرفته)

سوال مشابه: ثابت كنيد بي نهايت عدد وجود دارد كه بزرگترين مقسوم عليه n^4+1 بزرگتر از 2n^2 باشد
 

AHZolfaghari

Well-Known Member
ارسال ها
935
لایک ها
1,654
امتیاز
93
#70
پاسخ : ماراتن نظریه ی اعداد (سطح پیشرفته)

حقیقتش من تصور نمی کردم بچه های تشنه فعالیت هستند در سایت !!! به خاطر همین در چند جا هم گفته بودم که دیگه قسمت ریاضی از رونق افتاده !
اما دیروز که یکی از رفقا دو تاپیک ماراتن جبر و اعداد گذاشتند خیلی خوب استقبال شد هرچند جناب math بستند و صدالبته به حق هم بستند !
اما ادامش اینجا باشه که به قوانین هم عمل بشه دیگه ! فقط اینکه سوال ها خیلی سنگین نباشه در حد مرحله دو یا یه خرده بیشتر باشه بهتره تا آمادگی مرحله دو رو ایجاد کنه در بچه ها !
شاید مثل قدیما نشه ، اما میشه به یه جایی خوبی رسوند ! اگه این عطش در درون همه باقی بمونه ! به امید خدا یاعلی استارت کار رو بزنیم !
 

ash1374

New Member
ارسال ها
253
لایک ها
422
امتیاز
0
#71
پاسخ : ماراتن نظریه ی اعداد (سطح پیشرفته)

که در آن r عددی طبیعی و p عددی اول است که

عددی طبیعی است که
مکعب کامل است. ثابت کنید اگر
عددی طبیعی و کوچکتر از
باشد آنگاه
مکعب کامل نیست.

باشد که استقبال کنند...
 
ارسال ها
337
لایک ها
82
امتیاز
0
#72
پاسخ : ماراتن نظریه ی اعداد (سطح پیشرفته)

جواب رو لطفا بذارید
 

mahmoud20ni

New Member
ارسال ها
18
لایک ها
13
امتیاز
3
#74
پاسخ : ماراتن نظریه ی اعداد (سطح پیشرفته)

برای اثبات حکم کافی است نشان دهیم که معادله ی روبرو با شرط n<m جواب ندارد
. اگر این معادله دارای جواب باشد انگاه میتوان فرض کرد که n , p نسبت به هم اولند در این صورت خواهیم داشت :
و چون p عددی به شکل 3k+2 است و عوامل اول
یا 3 و یا به شکل 3k+2 هستند پس خواهیم داشت :
یعنی
اکنون اگر در معادله اصلی به جای a عبارت بدست امده را قرار دهیم معادله به شکل زیر در می اید :
یا به طور مرتب تر :
که در این صورت خواهیم داشت :
که تناقض است و در نتیجه معادله اولیه جواب ندارد .
 

ash1374

New Member
ارسال ها
253
لایک ها
422
امتیاز
0
#75
پاسخ : ماراتن نظریه ی اعداد (سطح پیشرفته)

سوال بعدی : عدد
را کامل گویند اگر
که
مجموع مقسوم علیه های عدد طبیعی
است(مثلا 6 و 28 اعدادی کامل هستند). فرض کنید
عددی کامل باشد و
تجزیه ی این عدد به عوامل اولش باشد طوری که
. نشان دهید
زوج است!
 
آخرین ویرایش توسط مدیر

mahmoud20ni

New Member
ارسال ها
18
لایک ها
13
امتیاز
3
#76
پاسخ : ماراتن نظریه ی اعداد (سطح پیشرفته)

اگر
فرد باشد خواهیم داشت :
که با توجه به زوج بودن 1+
خواهیم داشت
که با توجه به عبارت بالا میتوان نتیجه گرفت :
و چون
کوچکترین عامل اول n میباشد پس
و
که اگر حاصل را در عبارت ابتدایی جاگزین کنیم و پس از کمی ساده کردن خواهیم داشت :
که با مقایسه دو طرف این تساوی به سادگی نتیجه میشود که :
که به سادگی میتوان نتیجه گرفت که این نابرابری به طوراکید جواب ندارد و تنها حالت تساوی میتوان رخ دهد که در این حالت
و همچنین در این حالت n نمیتواند عامل اولی غیر از 2 و3 داشته باشد و به عبارتی n=6 که با n>6 در تناقض است و فرض اولیه غلط بوده و
عددی زوج است .
 

ash1374

New Member
ارسال ها
253
لایک ها
422
امتیاز
0
#77
پاسخ : ماراتن نظریه ی اعداد (سطح پیشرفته)

همه ی
های طبیعی را بیابید که


لطفاً هر کس حل کرد سوال بعدی رو هم خودش بگذاره و ماراتن رو خودتون به دست بگیرید.
 

darya.f

New Member
ارسال ها
182
لایک ها
114
امتیاز
0
#78
پاسخ : ماراتن نظریه ی اعداد (سطح پیشرفته)

همه ی
های طبیعی را بیابید که


لطفاً هر کس حل کرد سوال بعدی رو هم خودش بگذاره و ماراتن رو خودتون به دست بگیرید.
a,bهم زوجىت اندو با بررسى معلومه که هردو زوج اند پس
 

AHZolfaghari

Well-Known Member
ارسال ها
935
لایک ها
1,654
امتیاز
93
#79
پاسخ : ماراتن نظریه ی اعداد (سطح پیشرفته)


a,bهم زوجىت اندو با بررسى معلومه که هردو زوج اند پس
تیکه آخر چی شد که نتیجه گرفتید 'a برابر یک هستش ؟
 

REZA 73

Active Member
ارسال ها
139
لایک ها
184
امتیاز
43
#80
پاسخ : ماراتن نظریه ی اعداد (سطح پیشرفته)

میدانیم که a ,b هر دو زوج هستند. فرض کنید a=2m و b=2nدر نتیجه :

اما عبارت داخل قدرمطلق بزرگتر مساوی 4 هست. چون به هنگ 3 و 2 به ترتیب 1 و 0 هست.

اما به ازای هر n مقدار n^2 کوچکتر از 3 به توان n هست. در نتیجه :

پس تنها مقدار a برابر 4 است. بقیه فک کنم دیگه ساده باشه. یه سری چک کردن سادس چون n هم محدود میشه.
 
بالا